0% found this document useful (0 votes)
303 views49 pages

2023 AMC 10B Solutions

美国数学竞赛AMC10的2023年 B卷题解,含多种解法和讲解

Uploaded by

panda.yh0207
Copyright
© © All Rights Reserved
We take content rights seriously. If you suspect this is your content, claim it here.
Available Formats
Download as PDF, TXT or read online on Scribd
0% found this document useful (0 votes)
303 views49 pages

2023 AMC 10B Solutions

美国数学竞赛AMC10的2023年 B卷题解,含多种解法和讲解

Uploaded by

panda.yh0207
Copyright
© © All Rights Reserved
We take content rights seriously. If you suspect this is your content, claim it here.
Available Formats
Download as PDF, TXT or read online on Scribd

Problem_1

The following problem is from both the 2023 AMC 10B #1 and 2023 AMC 12B #1, so both problems redirect to this page.

Problem
Mrs. Jones is pouring orange juice into four identical glasses for her four sons. She fills the first three glasses completely but runs

out of juice when the fourth glass is only full. What fraction of a glass must Mrs. Jones pour from each of the first three glasses
into the fourth glass so that all four glasses will have the same amount of juice?

Solution 1
The first three glasses each have a full glass. Let's assume that each glass has "1 unit" of juice. It won't matter exactly how much
juice everyone has because we're dealing with ratios, and that wouldn't affect our answer. The fourth glass has a glass that is one

third. So the total amount of juice will be . If we divide the total amount of juice by 4, we get , which

should be the amount of juice in each glass. This means that each of the first three glasses will have to contribute

to the fourth glass.

~Sir Ian Seo the Great & lprado

Solution 2 (unnecessary numerical values)

Given that the first three glasses are full and the fourth is only full, let's represent their contents with a common denominator,

which we'll set as 6. This makes the first three glasses full, and the fourth glass full.

To equalize the amounts, Mrs. Jones needs to pour juice from the first three glasses into the fourth. Pouring from each of the

first three glasses will make them all full. Thus, all four glasses will have the same amount of juice. Therefore, the answer is

~Ishaan Garg

Solution 3
We let denote how much juice we take from each of the first children and give to the th child.

We can write the following equation: , since each value represents how much juice each child (equally) has

in the end. (Each of the first three children now have juice, and the fourth child has more juice on top of their initial .)

Solving, we see that

~Technodoggo
Problem_2
The following problem is from both the 2023 AMC 10B #2 and 2023 AMC 12B #2, so both problems redirect to this page.

Problem
Carlos went to a sports store to buy running shoes. Running shoes were on sale, with prices reduced by on every pair of
shoes. Carlos also knew that he had to pay a sales tax on the discounted price. He had dollars. What is the original
(before discount) price of the most expensive shoes he could afford to buy?

Solution 1 (easy)
We can create the equation:

using the information given. This is because x, the original price, got reduced by 20%, or multiplied by 0.8, and it also got multiplied
by 1.075 on the discounted price. Solving that equation, we get

~lprado

Solution 2 (One-Step Equation)


The discounted shoe is off the original price. So that means . There is also a sales tax charge, so
. Now we can set up the equation , and solving that we get ~
kabbybear

Solution 3
Let the original price be dollars. After the discount, the price becomes dollars. After tax, the price becomes

dollars. So, ,

~Mintylemon66

~ Minor tweak:Multpi12

Solution 4
We can assign a variable to represent the original cost of the shoes. Next, we set up the equation
. We can solve this equation for and get .
~vsinghminhas

Solution 5 (Intuition and Guessing)


We know the discount price will be 5/4, and 0.075 is equal to 3/40. So we look at answer choice , see that the discoutn price

will be 40, and with sales tax applied it will be 43, so the answer choice is .

Solution 6 (Not really a solution, DON'T DO THIS ON A REAL TEST)


Open up a coding IDE and use Python to solve this problem. Python code:

budget = 43.0
discount_percentage = 20.0
sales_tax_percentage = 7.5
discounted_price = budget / 1.075 / 0.8
print(f"${discounted_price:.2f}")

~Ishaan Garg
Problem_3
The following problem is from both the 2023 AMC 10B #3 and 2023 AMC 12B #3, so both problems redirect to this page.

Problem
A right triangle is inscribed in circle , and a right triangle is inscribed in circle . What is the ratio
of the area of circle to the area of circle ?

Solution 1
Because the triangle are right triangles, we know the hypotenuses are diameters of circles and . Thus, their radii are 2.5 and
6.5 (respectively). Square the two numbers and multiply to get and as the areas of the circles. Multiply 4 on

both numbers to get and . Cancel out the , and lastly, divide, to get your answer

~[Link]

Solution 2
Since the arc angle of the diameter of a circle is degrees, the hypotenuse of each these two triangles is respectively the
diameter of circles and .

Therefore the ratio of the areas equals the radius of circle squared : the radius of circle squared the diameter of
circle , squared : the diameter of circle , squared the diameter of circle , squared: the diameter of circle ,

squared

~Mintylemon66

Solution 3
The ratio of areas of circles is the same as the ratios of the diameters squared (since they are similar figures). Since this is a right

triangle the hypotenuse of each triangle will be the diameter of the circle. This yields the expression

~vsinghminhas
Problem_4
The following problem is from both the 2023 AMC 10B #4 and 2023 AMC 12B #4, so both problems redirect to this page.

Problem
Jackson's paintbrush makes a narrow strip with a width of millimeters. Jackson has enough paint to make a strip meters
long. How many square centimeters of paper could Jackson cover with paint?

Solution 1
millimeters is equal to centimeters. meters is centimeters. The answer is , so the answer is
.

~[Link]

Solution 2 (Standard Form)


millimeters can be represented as centimeters. meters is centimeters. Multiplying out these
results in , which is making the answer .

~[Link]
Problem_5
Problem
Maddy and Lara see a list of numbers written on a blackboard. Maddy adds to each number in the list and finds that the sum of
her new numbers is . Lara multiplies each number in the list by and finds that the sum of her new numbers is also . How
many numbers are written on the blackboard?

Solution
Let there be numbers in the list of numbers, and let their sum be . Then we have the following

From the second equation, . So,

~Mintylemon66 (formatted atictacksh)

Solution 2
Let where represents the th number written on the board. Lara's multiplied each number by , so her
sum will be . This is the same as . We are given

this quantity is equal to , so the original numbers add to . Maddy adds to each of the terms which yields,

. This is the same as the sum of the original series plus . Setting
this equal to ,

~vsinghminhas
Problem_6
Problem
Let , and for . How many terms in the sequence
are even?

Solution 1
We calculate more terms:

We find a pattern: if is a multiple of , then the term is even, or else it is odd. There are multiples of

from to .

~Mintylemon66

Solution 2
Like in the other solution, we find a pattern, except in a more rigorous way. Since we start with and , the next term is .

We start with odd, then odd, then (the sum of odd and odd) even, (the sum of odd and even) odd, and so on. Basically the pattern
goes: odd, odd, even, odd odd, even, odd, odd even…

When we take we get with a remainder of one. So we have full cycles, and an extra odd at the end.

Therefore, there are evens.

~e_is_2.71828

~SpreadTheMathLove

~e_is_2.71828
Problem_7
Problem
Square is rotated clockwise about its center to obtain square , as shown below.

What is the degree measure of ?

Solution 1
First, let's call the center of both squares . Then, , and since , .
Then, we know that bisects angle , so . Subtracting from , we get

~jonathanzhou18

Solution 2
First, label the point between and point and the point between and point . We know that and
that . Subtracting and from , we get that is . Subtracting from , we get that
. From this, we derive that . Since triangle is an isosceles triangle, we get that
. Therefore, . The answer is .

~yourmomisalosinggame (a.k.a. Aaron)

Solution 3
Call the center of both squares point , and draw circle such that it circumscribes the squares. and

, so . Since is inscribed in arc , .


~hpotter2021

Solution 4
Draw : we want to find . Call the point at which and intersect. Reflecting over , we
have a parallelogram. Since , angle subtraction tells us that two of the angles of the parallelogram are .
The other two are equal to (by properties of reflection).

Since angles on the transversal of a parallelogram sum to , we have , yielding

-Benedict T (countmath1)
Problem_8
Problem
What is the units digit of ?

Solution

(mod 10). ~andliu766

Solution 2 (Digit Cycles)


Note that the units digit will be the same regardless of the tens, hundreds, and thousands digits, so we can simplify this problem to
finding the last digit of . We can find the units digit of , by listing the units digits of the first few powers of
two, and trying to find a pattern.

As we can see the units digits of powers of two repeat after every four iterations. Now we know the units digit of is and
the units digit of . Similarly we can find the last digits of powers of three repeat after
every four, so the units digit of is . Adding these together, the ones digit is the same as the ones digit of
which is .

~vsinghminhas

Solution 3
When looking at the units digit patterns of the powers of , we see that

, units digit

, units digit

, units digit

, units digit

, units digit

And the pattern repeats. This pattern will apply for the powers of as well, since the units digit of is . We can find the
pattern for the powers of too. The pattern follows with units digits, , , , , , , ... Similarly, the units digit of will follow
the same pattern as the powers of .

Both of these powers cycle in groups of . When diving by , we get remainder , meaning complete cycles; or
the power being a multiple of , times, and extra. So the units digit of is . divided by is
reminder , which means complete cycles, or the power being a multiple of , times, and extra. So the units digit of
is .

We only need to find the units digit in the end, so we just add those already found units digits, to get a new units digit of .
Therefore the answer is
~mk
Problem_9
Problem
The numbers and are a pair of consecutive postive squares whose difference is . How many pairs of consecutive positive
perfect squares have a difference of less than or equal to ?

Solution 1
Let x be the square root of the smaller of the two perfect squares. Then,
. Thus, . So there are
numbers that satisfy the equation.

~andliu766

A very similar solution offered by ~[Link] and ~DarkPheonix has been combined with Solution 1.

Minor corrections by ~milquetoast

Note from ~milquetoast: Alternatively, you can let be the square root of the larger number, but if you do that, keep in mind that
must be rejected, since cannot be .

Solution 2
The smallest number that can be expressed as the difference of a pair of consecutive positive squares is , which is .
The largest number that can be expressed as the difference of a pair of consecutive positive squares that is less than or equal to
is , which is . Since these numbers are in the form , which is just .
These numbers are just the odd numbers from 3 to 2023, so there are such numbers. The

answer is .

~Aopsthedude
Problem_10
The following problem is from both the 2023 AMC 10B #10 and 2023 AMC 12B #5, so both problems redirect to this page.

Problem
You are playing a game. A rectangle covers two adjacent squares (oriented either horizontally or vertically) of a grid
of squares, but you are not told which two squares are covered. Your goal is to find at least one square that is covered by the
rectangle. A "turn" consists of you guessing a square, after which you are told whether that square is covered by the hidden
rectangle. What is the minimum number of turns you need to ensure that at least one of your guessed squares is covered by the
rectangle?

Solution 1
Notice that the square grid has a total of possible rectangles.

Suppose you choose the middle square for one of your turns. The middle square is covered by rectangles, and each of the
remaining squares is covered by a maximum of uncounted rectangles. This means that the number of turns is at least

Now suppose you don't choose the middle square. The squares on the middle of the sides are covered by at most 3 uncounted
rectangles, and the squares on the corners are covered by at most 2 uncounted rectangles. In this case, we see that the least
number of turns needed to account for all 12 rectangles is To prove that choosing only side squares indeed does
cover all 12 rectangles, we need to show that the 3 rectangles per square that cover each side square do not overlap. Drawing the
rectangles that cover one square, we see they form a shape and they do not cover any other side square. Hence, our answer is

Solution 2
First, note that since the rectangle covers 2 squares, we only need to guess squares that are not adjacent to any of our other
guesses. To minimize the amount of guesses, each of our guessed squares should try to touch another guess on one vertex and
one vertex only. There are only two ways to do this: one with guesses, and one with . Since the problem is asking for the
minimum number, the answer is .

~yourmomisalosinggame (a.k.a. Aaron)

Solution 3
Since the hidden rectangle can only hide two adjacent squares, we may think that we eliminate 8 squares and we're done, but think
again. This is the AMC 10, so there must be a better solution (also note that every other solution choice is below 8 so we're
probably not done) So, we think again, we notice that we haven't used the adjacent condition, and then it clicks. If we eliminate the
four squares with only one edge on the boundary of the 9x9 square. We are left with 5 diagonal squares, since our rectangle cant be
diagonal, we can ensure that we find it in 4 moves. So our answer is :

~arrowskyknight22

Solution 4 (checkerboard)
The 3x3 grid can be colored like a checkerboard with alternating blacks and whites. Let the top left square be white, and the rest of
the squares alternate colors.

Each rectangle will always cover exactly 1 white square and 1 black square. You can ensure that at least one of your
guessed squares is covered by the rectangle by guessing either all the white squares only ( turns) or all the black squares only (
turns).

In our case, guessing all the black squares takes turns, which is less than guessing all the white squares.

~ CherryBerry

Solution 5 (Logic)
We realize that every rectangle must contain an edge and no more than one edge. There are a total of four edges so the
answer is . ~[Link]

See Also
Problem_11
Problem
Suzanne went to the bank and withdrew . The teller gave her this ammount using bills, bills, and bills, with
at least one of each denomination. How many different collections of bills could Suzanne have received?

Solution 1
We let the number of , , and bills be and respectively.

We are given that Dividing both sides by , we see that

We divide both sides of this equation by : Since and are integers, must also be an

integer, so must be divisible by . Let where is some positive integer.

We can then write Dividing both sides by , we have We divide by

here to get and are both integers, so is also an integer. must be divisible by , so we let

We now have . Every substitution we made is part of a bijection (i.e. our


choices were one-to-one); thus, the problem is now reduced to counting how many ways we can have and such that they
add to .

We still have another constraint left, that each of and must be at least . For , let We
are now looking for how many ways we can have

We use a classic technique for solving these sorts of problems: stars and bars. We have stars and groups, which implies

bars. Thus, the total number of ways is

~Technodoggo ~minor edits by lucaswujc

Solution 2
First, we note that there can only be an even number of dollar bills.

Next, since there is at least one of each bill, we find that the amount of dollar bills is between and . Doing some casework,
we find that the amount of dollar bills forms an arithmetic sequence: + + + + + .

Adding these up, we get .

~yourmomisalosinggame (a.k.a. Aaron)

Solution 3
Denote by , , the amount of $20 bills, $50 bills and $100 bills, respectively. Thus, we need to find the number of tuples
with that satisfy

First, this equation can be simplified as

Second, we must have . Denote . The above equation can be converted to


Third, we must have . Denote . The above equation can be converted to

Denote , and . Thus, the above equation can be written as

Therefore, the number of non-negative integer solutions is .

~Steven Chen (Professor Chen Education Palace, [Link])

Solution 4
To start, we simplify things by dividing everything by , the resulting equation is , and since the
problem states that we have at least one of each, we simplify this to . Note that since the total is odd,
we need an odd number of dollar bills. We proceed using casework.

Case 1: One dollar bill

, we see that can be or . Ways

Case 2: Three dollar bills

, like before we see that can be , so way.

Now we should start to see a pattern emerges, each case there is less way to sum to , so the answer is just ,

or

~andyluo

Solution 5
We notice that each $100 can be split 3 ways: 5 $20 dollar bills, 2 $50 dollar bills, or 1 $100 dollar bill.

There are 8 of these $100 chunks in total--take away 3 as each split must be used at least once.

Now there are five left--so we use stars and bars.

5 chunks, 3 categories or 2 bars. This gives us

~not_slay

Solution 6 (kind of bash)


Casework on if there is one 100. There are 6 ways. Casework on if there are two 100s. There are 5 ways. Notice that this continues

all the way until there are 6 100s. Our sum is , or . ~MC413551
Problem_12
The following problem is from both the 2023 AMC 10B #12 and 2023 AMC 12B #6, so both problems redirect to this page.

Problem
When the roots of the polynomial

are removed from the number line, what remains is the union of 11 disjoint open intervals. On how many of these intervals is
positive?

Solution 1
is a product of or 10 terms. When , all terms are , but because there is an even
number of terms. The sign keeps alternating . There are 11 intervals, so there are positives and 5

negatives.

(This solution makes no sense, it is correct by luck) ~SpencerD.

(The method is incorrect, but the answer is correct by chance. The actual alternating sign is
for all 11 intervals. We count 6 intervals being positive which is our answer,
.) ~Bread10

Solution 2
Denote by the interval for and the interval .

Therefore, the number of intervals that is positive is

~Steven Chen (Professor Chen Education Palace, [Link])

Solution 3
The roots of the factorized polynomial are intervals from numbers 1 to 10. We take each interval as being defined as the number
behind it. To make the function positive, we need to have an even number of negative expressions. Real numbers raised to even
powers are always positive, so we only focus on , , , , and . The intervals 1 and 2 leave 4
negative expressions, so they are counted. The same goes for intervals 5, 6, 9, and 10. Intervals 3 and 4 leave 3 negative
expressions and intervals 7 and 8 leave 1 negative expression. The solution is the number of intervals which is .
~[Link] ~DarkPheonix

Solution 4
We can use the turning point behavior at the roots of a polynomial graph to find out the amount of intervals that are positive.

First, we evaluate any value on the interval . Since the degree of is = =

, and every term in P(x) is negative, multiplying 55 negatives gives a negative value. So is a negative interval.

We know that the roots of P(x) are at . When the degree of the term of each root is odd, the graph of P(x) will pass
through the graph and change signs, and vice versa. So at , the graph will change signs; at , the graph will not, and
so on.

This tells us that the interval is positive, is also positive, is negative, is also negative, and so on, with
the pattern being .

The positive intervals are therefore , , , , , and , for a total of .

~nm1728

Solution 5
The expressions to the power of even powers are always positive, so we don't need to care about those. We only need to care about
. We need 0, 2, or 4 of the expressions to be negative. The 9 through 10
interval and 10 plus interval make all of the expressions positive. The 5 through 6 and 6 through 7 intervals make two of the
expressions negative. The 1 through 2 and 2 through 3 intervals make four of the expressions negative. There are
intervals.

~Aopsthedude

See Also
Problem_13
The following problem is from both the 2023 AMC 10B #13 and 2023 AMC 12B #9, so both problems redirect to this page.

Problem
What is the area of the region in the coordinate plane defined by

Solution 1
First consider, We can see that it's a square with radius 1 (diagonal ). The area of the square is

Next, we add one more absolute value and get This will double the square reflecting over x-axis.

So now we got 2 squares.

Finally, we add one more absolute value and get This will double the squares reflecting over y-
axis.

In the end, we got 4 squares. The total area is

~Technodoggo ~Minor formatting change: e_is_2.71828

Solution 2 (Graphing)
We first consider the lattice points that satisfy and . The lattice points satisfying these
equations are and By symmetry, we also have points
and when and . Graphing
and connecting these points, we form 5 squares. However, we can see that any point within the square in the middle does not
satisfy the given inequality (take , for instance). As noted in the above solution, each square has a diagonal for an area of

, so the total area is

~ Brian__Liu

Note
This problem is very similar to a past AIME problem (1997 P13)

~ CherryBerry

Solution 3 (Logic)
The value of and can be a maximum of 1 when the other is 0. Therefore the value of and range from -2 to 2. This forms

a diamond shape which has area which is

~ [Link] ~ DarkPheonix

See Also
Problem_14
Problem
How many ordered pairs of integers satisfy the equation ?

Solution 1
Clearly, is 1 solution. However there are definitely more, so we apply to get this:

This basically say that the product of two consecutive numbers must be a perfect square which is practically
impossible except or . gives . gives .

~Technodoggo ~minor edits by lucaswujc

Solution 2
Case 1: .

In this case, .

Case 2: .

Denote . Denote and . Thus, .

Thus, the equation given in this problem can be written as

Modulo , we have . Because , we must have . Plugging this into the


above equation, we get . Thus, we must have and .

Thus, there are two solutions in this case: and .

Putting all cases together, the total number of solutions is .

~Steven Chen (Professor Chen Education Palace, [Link])

Solution 3 (Discriminant)
We can move all terms to one side and wrote the equation as a quadratic in terms of to get

The discriminant of this quadratic is

For to be an integer, we must have be a perfect square. Thus, either is a perfect square or
and . The first case gives , which result in the equations and ,
for a total of two pairs: and . The second case gives the equation , so it's only pair is . In total,
the total number of solutions is .

~A_MatheMagician
Problem_15
Problem
What is the least positive integer such that is a perfect square?

Solution 1
Consider 2 -- there are odd number of 2's in (We're not counting 3 2's in 8, 2 3's in 9, etc).

There are even number of 3's in ...etc,

So, we can reduce our original expression to

~Technodoggo ~minor edits by lucaswujc

Solution 2
Perfect squares have all of the powers in their prime factorization even. To evaluate we get the following:

Taking all powers we get:

Simplifying again, we finally get:

To make all the powers left even, we need to multiply by which is .

~[Link]

Solution 3
We can prime factorize the solutions: A = B= C= D= E=

We can immediately eliminate B, D, and E since 13 only appears in , so is a perfect square. Next, we can test if 7 is possible (and if it is not we can use
process of elimination). 7 appears in to and 14 appears in to . So, there is an odd amount of 7's since there are 10 7's from to and 3 7's from to since 7 appears in
14 once, and which is odd. So we need to multiply by 7 to get a perfect square. Since 30 is not a divisor of 7, our answer is 70 which is .

~aleyang

Solution 4
First, we note that , . So, . Simplifying the
whole sequence and cancelling out the squares, we get . Prime factoring and cancelling out the squares, the only numbers that remain are and
. Since we need to make this a perfect square, . Multiplying this out, we get .

~yourmomisalosinggame (a.k.a. Aaron) & ~Technodoggo (add more examples)

Solution 5 (Bashy method)


We know that a perfect square must be in the form where are nonnegative integers, and is the largest and prime factor of our
square number.

Let's assume . We need to prime factorize and see which prime factors are raised to an odd power. Then, we can multiply one factor each of prime number
with an odd number of factors to . We can do this by finding the number of factors of , , , , , and .

Case 1: Factors of

We first count factors of in each of the factorials. We know there is one factor of each in and , two in and , and so on until we have factors of in . Adding them all up, we
have .

Now, we count factors of in each of the factorials. We know there is one factor of each in , , , and , two in , , , and , and so on until we have factors of in .
Adding them all up, we have .

Now we count factors of in each of the factorials. Using a similar method as above, we have a sum of .

Now we count factors of in each of the factorials. Using a similar method as above, we have a factor of in , so there is factor of .

Adding all the factors of , we have . Since is odd, has one factor of .

Case 2: Factors of

We use a similar method as in case 1. We first count factors of . We obtain the sum .

We count factors of . We obtain the sum .

Adding all the factors of , we have . Since is even, has factors of .

Case 3: Factors of
We count the factors of : . Since is odd, has one factor of .

Case 4: Factors of

We count the factors of : . Since is odd, has one factor of .

Case 5: Factors of

We count the factors of : . Since is even, has factors of .

Case 6: Factors of

We count the factors of : . Since is even, has factors of .

Multiplying out all our factors for , we obtain .

~arjken
Problem_16
Problem
Define an to be a positive integer of or more digits where the digits are strictly increasing moving left to right. Similarly,
define a to be a positive integer of or more digits where the digits are strictly decreasing moving left to right. For
instance, the number is an upno and is a downno. Let equal the total number of and let equal the
total number of . What is ?

Solution 1
First, we know that is greater than , since there are less upnos than downnos. To see why, we examine what determines an
upno or downno.

We notice that, given any selection of unique digits (notice that "unique" constrains this to be a finite number), we can construct a
unique downno. Similarly, we can also construct an upno, but the selection can not include the digit since that isn't valid.

Thus, there are total downnos and total upnos. However, we are told that each upno or downno must be at least digits, so
we subtract out the -digit and -digit cases.

For the downnos, there are -digit cases, and for the upnos, there are -digit cases. There is -digit case for both upnos
and downnos.

Thus, the difference is

~Technodoggo ~minor edits by lucaswujc

Solution 2
Since Upnos do not allow 0s to be in their first -- and any other -- digit, there will be no zeros in any digits of an Upno. Thus, Upnos
only contain digits [1,2,3,4,5,6,7,8,9].

Upnos are 2 digits in minimum and 9 digits maximum (repetition is not allowed). Thus the total number of Upnos will be (9C2)+
(9C3)+(9C4)+...+(9C9), since every selection of distinct numbers from the set [1,2,3,4,5,6,7,8,9] can be arranged so that it is an
Upno. There will be (9C2) 2 digit Upnos, (9C3) 3 digit Upnos and so on.

Thus, the total number of Upnos will be (9C2)+(9C3)+(9C4)+...+(9C9) = 2^9-(9C0)-(9C1) = 512 - 10 = 502.

Notice that the same combination logic can be done for Downnos, but Downnos DO allow zeros to be in their last digit. Thus, there
are 10 possible digits [0,1,2,3,4,5,6,7,8,9] for Downnos.

Therefore, it is visible that the total number of Downnos are (10C2)+(10C3)+(10C4)+...+(10C10) = 2^10-(10C0)-(10C10) = 1024 - 11
= 1013.

Thus abs(#Upno-#Downno) = abs(1013-502) = 511.

~yxyxyxcxcxcx

Solution 3
Note that you can obtain a downo by reversing an upno (like is an upno, and you can obtain ). So, we need to find the
amount of downos that end with 0 since if you 'flip' the numbers, the upno starts with a 0. We can find the cases that end with a 0:

to get 512. However, 0 itself is not a valid case (since it has 1 digit) so we subtract 1. Our answer is 511.

-aleyang
-ap246(LaTeX)

Solution 4 (Educated Guess)


First, note that the only that are not contained by the set of is every that ends in .

Next, listing all the two digits , we find that the answer is more than 9, since there are more digits to be tested and
there are 9 two digit . This leaves us with or .

Next, we notice that all the possibilities for through digit ending in pair up with one another, as the possibilities
are equal (e.g. possibilities for digits = possibilities for digits, etc.).

This leaves us with one last possibility, the ten digit .

Since all the previous possibilities form an even number, adding one more possibility will make the total odd. Therefore, we need to
choose the odd number from the set .

Our answer is .

~yourmomisalosinggame (a.k.a. Aaron)

Solution 5
We start by calculating the number of upnos. Suppose we are constructing an upno of digits such that . An upno can't

start with a " ", so there are digits to choose from. There are ways to choose an upno with digits. This is because for

each combination of digits, only one combination can form an upno. Therefore, for , the total number of upnos is

Similarly, the digits of a downo of digits can be chosen among 10 digits to choose from, since can be a digit of the downo as
the last digit. Thus, the number of downos is

Thus,

~rnatog337

Solution 6
Note that since the only way upno and downo can be different is if the downo ends in , so that the corresponding upno cannot
exist. Therefore we just have to calculate the number to downos that end with .This ends up being (2 options of whether or not
the number exists in the upnoto or not) However we then need to subtract by 1 the needs to have 2 digits and 0
would have been a possible without this restiction (although we would still need to remove it since 0 would

have been both an upno and downo)giving us , or


Problem_17
The following problem is from both the 2023 AMC 10B #17 and 2023 AMC 12B #13, so both problems redirect to this page.

Problem
A rectangular box has distinct edge lengths , , and . The sum of the lengths of all edges of is , the areas of all

faces of is , and the volume of is . What is the length of the longest interior diagonal connecting two vertices of ?

Solution 1 (algebraic manipulation)

We can create three equations using the given information.

We also know that we want because that is the length that can be found from using the Pythagorean Theorem.

We cleverly notice that . We know that and

, so . So our answer is

Interestingly, we don't use the fact that the volume is .

~lprado

~Technodoggo

~minor edits and add-ons by lucaswujc


~andliu766

Solution 2 (vieta's)
We use the equations from Solution 1 and manipulate it a little:

Notice how these are the equations for the vieta's formulas for a polynomial with roots of , , and . Let's create that polynomial.

It would be . Multiplying each term by 4 to get rid of fractions, we get

. Notice how the coefficients add up to . Whenever this happens, that means that is a
factor and that 1 is a root. After using synthetic division to divide by , we get
. Factoring that, you get . This means that this polynomial factors to
and that the roots are , , and . Since we're looking for , this is equal to

~lprado

Solution 3 (Cheese Method)


Incorporating the solution above, we know = . The side lengths are larger than
(a unit cube). The side length of the interior of a unit cube is , and we know that the side lengths are larger than , so
that means the diagonal has to be larger than , and the only answer choice larger than

~kabbybear

Note that the real number is around . Option is also greater than meaning there are two options greater than
. Option is an integer so educationally guessing we arrive at answer

~atictacksh

See Also
Problem_18
The following problem is from both the 2023 AMC 10B #18 and 2023 AMC 12B #15, so both problems redirect to this page.

Problem
Suppose , , and are positive integers such that

Which of the following statements are necessarily true?

I. If or or both, then .

II. If , then or or both.

III. if and only if .

Solution 1 (Guess and check + Contrapositive)


We examine each of the conditions.

The first condition is false. A simple counterexample is and . The corresponding value of is .
Clearly, and , so condition would imply that However,
is clearly not (they share a common factor of ). Obviously, condition is false, so we can rule out choices
and .

We are now deciding between the two answer choices and . What differs between them is the validity of condition , so it
suffices to simply check .

We look at statement 's contrapositive to prove it. The contrapositive states that if and
, then In other words, if shares some common factor that is not with and
shares some common factor that is not with , then also shares a common factor with . Let's say that ,
where is a factor of not equal to . (So is the common factor.)

We can rewrite the given equation as We can express as , for


some positive integer (this can be ). We can factor out to get

We know that all values in this equation are integers, so must be divisible by . Since is a factor of , must also be a
factor of , a multiple of . Therefore, we know that shares a common factor with (which is ), so
. This is what states, so therefore is true.

Thus, our answer is ~Technodoggo

Solution 2
The equation given in the problem can be written as

A counter example is and . Thus, .

First, we prove the ``if part.

Suppose and . However, .


Thus, must be divisible by at least one factor of 210. W.L.O.G, we assume is divisible by 2.

Modulo 2 on Equation (1), we get that . This is a contradiction with the condition that . Therefore, the ``if
part in Statement III is correct.

Second, we prove the ``only if part.

Suppose . Because , there must be one factor of 14 or 15 that divides . W.L.O.G, we


assume there is a factor of 14 that divides . Because , we have . Modulo on
Equation (1), we have .

Because , we have .

Analogously, we can prove that .

This is simply a special case of the ``only if part of Statement III. So we omit the proof.

All analyses above imply

~Steven Chen (Professor Chen Education Palace, [Link])

Solution 3 (Answer Choices)


It can easily be shown that statement I is false (a counterexample would be ), meaning the only viable
answer choices are D and E. Since both of these answer choices include statement III, this means III is true. Since III is true, this
actually implies that statement II is true, as III is just a stronger version of statement II (or it's contrapositive, to be precise).
Therefore the answer is

~SpencerD. ~edited by A_MatheMagician

See Also
Problem_19
Problem
Sonya the frog chooses a point uniformly at random lying within the square in the coordinate plane and hops to that point. She then randomly chooses a distance uniformly at
random from and a direction uniformly at random from {north, south, east, west}. All he choices are independent. She now hops the distance in the chosen direction. What is the probability
that she lands outside the square?

Solution 1
WLOG, we assume Sonya jumps units every time, since that is her expected value.

If Sonya is within blocks of an edge, she can jump off the board. Let us examine the region that is at most blocks from exactly one edge.

If Sonya starts in this region, she has a chance of landing outside (there's exactly one direction she can hop to get out). The total area of this region is For this region,

Sonya has a chance, so we multiply by to get

If Sonya is in one of the corner squares, she can go two directions to get out, so she has a chance to get out. The total area is , so this region yields

Adding the two, we get . This is out of square units of area, so our answer is thus

~Technodoggo

Solution 2
Since all the actions are independent, we can switch the orders. Let Sonya choose the direction first. And the problem is symmetric, so we consider just one direction. WLOG, let's say she choose

. When she first pick the location, she'll have to be within 1 unit of the axis to have a chance to jump out of the boundary southward. That's . With in that region, the expected y

coordinate would be 0.5 which is 0.5 unit from the boundary (x-axis). Now, the jumping distance required to jump out of the boundary on average has to be greater than 0.5. That's another . So

the final probability is .

~Technodoggo

Solution 3
We denote by the frog's initial coordinates. We denote by the direction to hop. We denote by the hopping distance. In this analysis, we say that the frog wins if
landing outside the square.

We have

$$ (Error compiling LaTeX. Unknown error_msg)\begin{align*} P \left( win \right) & = \sum_{k \in \left\{ n, e, s, w \right\}} P\left( win | k \right) P \left( k \right) \\ & = P \left( win | k = w \right)
\sum_{k \in \left\{ n, e, s, w \right\}} P \left( k \right) \\ & = P \left( win | k = w \right) \\ & = \int_{y=0}^1 P \left( win | k = w, y \right) dy \\ & = P \left( win | k = w, y = 0 \right) \\ & = P \left( win | k = w,
y = 0, x \in [0,1] \right) P \left( x \in [0,1] \right) + P \left( win | k = w, y = 0, x \in (1,6] \right) P \left( x \in (1,6] \right) \\ & = P \left( win | k = w, y = 0, x \in [0,1] \right) \cdot \frac{1}{6} + 0 \cdot

\frac{5}{6} \\ & = \frac{1}{6} P \left( win | k = w, y = 0, x \in [0,1] \right) \\ & = \frac{1}{6} \int_{x=0}^1 \int_{z=x}^1 dz dx \\ & = \frac{1}{6} \cdot \frac{1}{2} \\ & = \boxed{\textbf{(B) }}.

\end{align*}$$ (Error compiling LaTeX. Unknown error_msg) The second equality above follows from symmetry that are all the same for all . The fifth
equality above follows from symmetry that are all the same for all .

~Steven Chen (Professor Chen Education Palace, [Link])

Solution 4
We can notice that Sonya can only jump out if she first picks a point that is at most 1 unit from the border. Let's separate this region into 4 different strips each with overlapping corners. Since
each strip is exactly the same, let's first consider the probability of Sonya leaving the square given that she first lands on the top strip. Using expected value, one can get that the probability that

Sonya gets the distance needed to leave the square is . Now the probability she gets the direction needed (north) is . Now with the probability of landing in the strip being , we get the

probability that she lands on the strip and leaves the square to be Since there are four strips, we add the probabilities giving a final answer of
~Daniel C (Minor edits by Ishaan Garg)

Solution 5
We know that Sonya can only jump out if she is within 1 unit of the border. We can calculate the probability that Sonya can jump out.

The total area of the colored regions is , so the probability that Sonya lands in a colored region is . We can calculate the probability that Sonya gets out of each type of region.

Case 1: Sonya chooses a blue region.

The probability that Sonya chooses a blue region is . One direction can let her out, so the probability that she chooses the right one is . Finally, the probability that Sonya chooses a

distance to get her out is . So, the probability that she chooses a blue region and gets out is .

Case 2: Sonya choose a red region.

The probability that Sonya chooses a red region is . Two directions can let her out, so the probability that she chooses one of them is . Finally, the probability that Sonya chooses a

distance to get her out is . So, the probability that she chooses a red region and gets out is .

So, the probability that Sonya gets out is

~ MathCactus0_0
Problem_20
Problem
Four congruent semicircles are drawn on the surface of a sphere with radius , as shown, creating a close curve that divides the
surface into two congruent regions. The length of the curve is . What is ?

Solution 1
There are four marked points on the diagram; let us examine the top two points and call them and . Similarly, let the bottom
two dots be and , as shown:

This is a cross-section of the sphere seen from the side. We know that , and by
Pythagorean therorem,

Each of the four congruent semicircles has the length as a diameter (since is congruent to and ), so

its radius is Each one's arc length is thus

We have of these, so the total length is , so thus our answer is


~Technodoggo

Solution 2
Assume , , , and are the four points connecting the semicircles. By law of symmetry, we can pretty confidently assume
that is a square. Then, , and the rest is the same as the second half of solution .

~jonathanzhou18

Solution 3
We put the sphere to a coordinate space by putting the center at the origin. The four connecting points of the curve have the
following coordinates: , , , .

Now, we compute the radius of each semicircle. Denote by the midpoint of and . Thus, is the center of the semicircle
that ends at and . We have . Thus, .

In the right triangle , we have .

Therefore, the length of the curve is

Therefore, the answer is .

~Steven Chen (Professor Chen Education Palace, [Link])

Solution 4

Note that each of the diameters are the chord of the sphere of a quarter arc. Thus, the semicircles diameter's length is . Thus,

the entire curve is . Therefore, the answer is . ~andliu766


Problem_21
The following problem is from both the 2023 AMC 10B #21 and 2023 AMC 12B #19, so both problems redirect to this page.

Problem
Each of balls is randomly placed into one of bins. Which of the following is closest to the probability that each of the bins
will contain an odd number of balls?

Important Clarification
Stars and Bars does not provide an exact probability. However, it does provide a good estimate for the approximate answer as on
average, the number of arrangements will be almost the same when each container has an odd # of balls or when each container
has an even # of balls. (similar binomial distributions)

Solution 1
Because each bin will have an odd number, they will have at least one ball. So we can put one ball in each bin prematurely. We then
can add groups of 2 balls into each bin, meaning we now just have to spread 1010 pairs over 3 bins. This will force every bin to

have an odd number of balls. Using stars and bars, we find that this is equal to . This is equal to . The

total amount of ways would also be found using stars and bars. That would be . Dividing

our two quantities, we get . We can roughly cancel to get . The 2 in the numerator and

denominator also cancels out, so we're left with .

~lprado

~AtharvNaphade ~eevee9406 ~Teddybear0629

Solution 2 (Solution 1 with more steps)


Suppose the numbers are , , and . First, we try to calculate the amount of ways for all three balls to be placed in a bin so the
number of balls in each bin is odd. and each bin has at least one ball because they are positive odd

numbers. Changing the equation, we see that . Let

, , and . Thus . We can also see that , , and

are all positive. Using the positive version of stars and bars, we get = choices.

Now, we want to find the total amount of cases. Using the non-negative version of stars and bars, we find that the total is

= .

Now we need to calculate / , which is just . Cancelling the twos, we get

. This is roughly equal to . The answer is .


~Aopsthedude

Solution 3
Having 2 bins with an odd number of balls means the 3rd bin also has an odd number. The probability of the first bin having an odd

number of balls is , since even and odd have roughly the same probability. The probability of the second bin having an odd

number of balls is also for the same reason. If both of these bins have an odd number of balls, the number of balls remaining for

the third bin is also odd. Therefore the probability is .

~Yash C

Solution 4
We first examine the possible arrangements for parity of number of balls in each box for balls.

If a denotes an even number and a denotes an odd number, then the distribution of balls for balls could be
or . With the insanely overpowered magic of cheese, we assume that each case is about
equally likely.

From , it is not possible to get to all odd by adding one ball; we could either get or . For the
other cases, though, if we add a ball to the exact right place, then it'll work.

For each of the working cases, we have possible slot the ball can go into (for , for example, the new ball must go in the

center slot to make ) out of the slots, so there's a chance. We have a chance of getting one of these working cases,

so our answer is

~pengf ~Technodoggo

Solution 5
2023 is an arbitrary large number. So, we proceed assuming that an arbitrarily large number of balls have been placed.

For an odd-numbered amount of balls case, the 3 bins can only be one of these 2 combinations:

( , , )

( )

Let the probability of achieving the case to be and any of the permutations to be
.

Because the amount of balls is arbitrarily large, even after another two balls are be placed.

There are two cases for which placing another two balls results in :

: The two balls are placed in the same bin ( )

: The two balls are placed in the two even bins ( )

So,
~Dissmo

Solution 6
We use the generating functions approach to solve this problem. Define .

We have

First, we set , , . We get

Second, we set , , . We get

Third, we set , , . We get

Fourth, we set , , . We get

Taking , we get

The last expression above is the number of ways to get all three bins with odd numbers of balls. Therefore, this happens with
probability
~Steven Chen (Professor Chen Education Palace, [Link])

Solution 7
Four even-odd splittings divides in to three, namely , , , and . Here if
we define a "move" as relocated one ball, then we will notice in each case, that a random "move" will be evenly likely to be one of the
other three splittings. Hence by Group theory (or by intuition), we will find the structure of this splitting is group, and it's
symmetric for all four elements in this Group.

Thus, no matter what is the initial starting point, four cases will be evenly likely to appear when repeated many times. The answer is

~Prof. Joker

Solution 8
Really simple way to solve it. To have 3 numbers that are all odd, you need to get odd for the first two bins, and the last one will
always be odd. There are 2023 ball, so the chance of having a odd number in the first bin is 1012/2023 and the chance of having
another odd is 1/2. 1012/2023 * 1/2 is closest to 1/4.

~Jack Bai(only 9 years old)

See Also
Problem_22
Problem
How many distinct values of 𝑥 satisfy , where denotes the largest integer less than or equal to ?

Solution 1(three cases)

First, let's take care of the integer case--clearly, only work. Then, we know that must be an integer. Set .

Now, there are two cases for the value of . Case 1:

There are no solutions in this case. Case 2:

This case provides the two solutions and as two more solutions. Our final answer is thus .

~wuwang2002

Solution 2
First, are trivial solutions

We assume from the shape of a parabola and the nature of the floor function that any additional roots will be near 2 and 1

We can now test values for :

We have . Solving, we have . We see that , so this solution is valid

We have . Solving, we have . , so this is not valid. We assume there are no more
solutions in the negative direction and move on to

We have . Solving, we have . We see that , so this solution is valid

We have . Solving, we have . , so this is not valid. We assume there are no more
solutions.

Our final answer is


~kjljixx

Solution 3
Denote . Denote . Thus, .

The equation given in this problem can be written as

Thus,

Because , we have . Thus,

If , so can be .

If , which we find has no integer solutions after finding the discriminant.

If , -> so can also be .

Therefore, , 2, 0, 3. Therefore, the number of solutions is .

~Steven Chen (Professor Chen Education Palace, [Link])

Solution 4(Quick)
A quadratic equation can have up to 2 real solutions. With the , it could also help generate another pair. We have to verify that
the solutions are real and distinct.

First, we get the trivial solution by ignoring the floor. , we get as our first pair of solutions.

Up to this point, we can rule out A,E.

Next, we see that This implies that must be an integer. We can guess and check as which yields

So we got 4 in total

~Technodoggo
Problem_23
Problem
An arithmetic sequence of positive integers has terms, initial term , and common difference . Carl wrote down all
the terms in this sequence correctly except for one term, which was off by . The sum of the terms he wrote was . What is
?

Solution 1
Since one of the terms was either more or less than it should have been, the sum should have been or

The formula for an arithmetic series is This can quickly be rederived by

noticing that the sequence goes , and grouping terms.

We know that or . Let us now show that is not possible.

If , we can simplify this to be Since every expression in

here should be an integer, we know that either and or and


The latter is not possible, since and The former is also impossible, as

Thus, .

We can factor as . Using similar reasoning, we see that can not be paired as and , but rather must
be paired as and with a factor of somewhere.

Let us first try Our equation simplifies to We know that so


we try the smallest possible value: This would give us (Indeed, this is the only
possible .)

There is nothing wrong with the values we have achieved, so it is reasonable to assume that this is the only valid solution (or all
solutions sum to the same thing), so we answer

For the sake of completeness, we can explore It turns out that we reach a contradiction in this case, so we are done.

~Technodoggo

Solution 2

There are terms, the th term is , summation is .

The summation of the set is . First, : its only possible factors are , and as said by
the problem, , so must be or . Let's start with . Then, , and this means ,
. Summing gives . We don't need to test any more cases, since the problem writes that all
are the same, so we don't need to test other cases.

-HIA2020
Problem_24
Problem
What is the perimeter of the boundary of the region consisting of all points which can be expressed as
with , and ?

Solution

Notice that this we are given a parametric form of the region, and is used in both and . We first fix and to , and graph
from :

Now, when we vary from to , this line is translated to the right units:
We know that any points in the region between the line (or rather segment) and its translation satisfy and , so we shade in the
region:

We can also shift this quadrilateral one unit up, because of . Thus, this is our figure:
The length of the boundary is simply ( can be obtained by Pythagorean theorem, since we have
side lengths and .). This equals

~Technodoggo
Problem_25
The following problem is from both the 2023 AMC 10B #25 and 2023 AMC 12B #25, so both problems redirect to this page.

Problem

A regular pentagon with area is printed on paper and cut out. The five vertices of the pentagon are folded into the center
of the pentagon, creating a smaller pentagon. What is the area of the new pentagon?

Solution 1

Let the original pentagon be centered at . The dashed lines represent the fold lines. WLOG, let's focus on vertex .

Since is folded onto , where is the intersection of and the creaseline between and . Note that
the inner pentagon is regular, and therefore similar to the original pentagon, due to symmetry.

Because of their similarity, the ratio of the inner pentagon's area to that of the outer pentagon can be represented by
Option 1: Knowledge

Remember that .

Option 2: Angle Identities

Let the inner pentagon be .


-Dissmo

Solution 2

Let and be the circumradius of the big and small pentagon, respectively. Let be the apothem of the smaller pentagon and
and be the areas of the smaller and larger pentagon, respectively.

From the diagram:


~Technodoggo

Solution 3
Interestingly, we find that the pentagon we need is the one that is represented by the intersection of the diagonals. Through similar

triangles and the golden ratio, we find that the side length ratio of the two pentagons is Thus, the answer is

. ~andliu766

Solution 4 (answer choices)


After drawing a decent diagram, we can see that the area of the inner pentagon is quite a bit smaller than half the area of the larger
pentagon.

Then, we can estimate the values of the answers and choose one that seems the closest to the smallest answer.

We know that , so we'll use for our estimations. The area of the original pentagon is
, so half of it is roughly .

A: clearly, this is wrong because it is greater than half the area of the pentagon.

B: This answer could be right.


C: This too.

D: This answer is wrong, as it assumes that the area of the inner pentagon is exactly half the area of the larger one.

E: This answer could be right.

But, from our diagram, assume that the area of the pentagon is significantly less than the area half of the larger pentagon, so we

choose the smallest answer choice, giving us . ~erics118

Supplement (Calculating sin54/cos36 from Scratch)


Method 1:

Construct golden ratio triangle with , and with ,


. WLOG, let , , .

Method 2: (Writing in progress...)

You might also like